Вы находитесь на странице: 1из 25

1

GATE ELECTRICAL ENGINEERING


SOLVED PAPER 2018

This Paper '’Gate Electrical Engineering Solved


Paper 2018’' is taken from our Book:

ISBN : 9789388026659
2

2018
Duration: 3 hrs Maximum Marks: 100

INSTRUCTIONS
1. There are a total of 65 questions carrying 100 marks.
2. This question paper consists of 2 sections, General Aptitude (GA) for 15 marks and the subject specific
GATE paper for 85 marks. Both these sections are compulsory.
3. The GA section consists of 10 questions. Question numbers 1 to 5 are of 1-mark each, while question numbers 6 to
10 are of 2-marks each. The subject specific GATE paper section consists of 55 questions, out of which question
numbers 11 to 35 are of 1-mark each, while question numbers 36 to 65 are of 2-marks each.
4. Questions are of Multiple Choice Question (MCQ) or Numerical Answer type. A multiple choice question will have
four choices for the answer with only one correct choice. For numerical answer type questions, the answer is a number
and no choices will be given.
5. Questions not attempted will result in zero mark. Wrong answers for multiple choice type questions will result in
1
NEGATIVE marks. For all 1 mark questions, mark will be deducted for each wrong answer. For all 2 marks questions,
3
2
mark will be deducted for each wrong answer..
3
6. There is NO NEGATIVE MARKING for questions of NUMERICAL ANSWER TYPE.

General Aptitude Test

QUESTIONS.1–5 CARRY ONE MARKS EACH. (a) 4, 8, 18 (b) 4, 10, 16


1. “Since you have gone off the ______, the _____ sand is (c) 4, 8, 28 (d) 8, 26, 28
likely to damage the car.” The words that best fill the blanks 5. Functions, F(a, b) and G(a, b) are defined as follows:
in the above sentence are. F(a, b) = (a – b)2 and G(a, b) = |a – b|, where |x| represents the
(a) course, coarse (b) course, course absolute value of x. What would be the value of G(F(1, 3),
(c) coarse, course (d) coarse, coarse G(1, 3))?
2. “A common misconception among writers is that sentence (a) 2 (b) 4 (c) 6 (d) 36
structure mirrors thought; the more ______ the structure, QUESTIONS 6 – 10 CARRY TWO MARKS EACH.
the more complicated the ideas.”
6. An e-mail password must contain three characters. The
The word that best fills the blank in the above Sentence is password has to contain one numeral from 0 to 9, one upper
(a) detailed (b) simple case and one lower case character from the English alphabet.
(c) clear (d) convoluted How many distinct passwords are possible?
3. Th e three roots of the equation f(x) = 0 are x = (a) 6,760 (b) 13,520
{–2, 0, 3}. What are the three values of x for which f (x – 3) = 0? (c) 40,560 (d) 1,05,456
(a) –5, –3, 0 (b) –2, 0, 3 7. In a certain code AMCF is written as EQGJ and NKUF is
written as ROYJ. How will DHLP be written in the code?
(c) 0, 6, 8 (d) 1, 3, 6
(a) RSTN (b) TLPH

4. For what values of k given below is


( k + 2)2 an integer?
(c) HLPT (d) XSVR
k-3
3
8. A class of twelve children has two more boys than girls. A
group of three children are randomly picked from this class
to accompany the teacher on a field trip. What is the i0
probability that the group accompanying the teacher
contains more girls than boys? +
L
325 o v0
(a) 0 (b) a
864
d
525 5 –
(c) (d)
864 12
9. A designer uses marbles of four different colours for his
designs. The cost of each marble is the same, irrespective of
the colour. The table below shows the percentage of marbles (a) v0 ³ 0 and i0 < 0 (b) v0 < 0 and i0 < 0
of each colour used in the current design. The cost of each (c) v0 ³ 0 and i0 ³ 0 (d) v0 < 0 and i0 ³ 0
marble increased by 25%. Therefore, the designer decided to 4. Four power semiconductor devices are shown in the figure
reduce equal numbers of marbles of each colour to keep the along with their relevant terminals. The device(s) that can
total cost unchanged. What is the percentage of blue marbles carry dc current continuously in the direction shown when
in the new design? gated approximately is (are)
A MT1 A
Blue Black Re d Yellow D
40% 25% 20% 15%
G
(a) 35.75 (b) 40.25 S
G K G MT2 G K
(c) 43.75 (d) 46.25
10. P, Q, R and S crossed a lake in a boat that can hold a maximum I
I I I
of two persons, with only one set of oars. The following
additional facts are available
(i) The boat held two persons on each of the three forward Thyristor Triac GTO MOSFET
trips across the lake and one person on each of the two (a) Triac only (b) Triac and MOSFET
return trips. (c) Triac and GTO (d) Thyristor and Triac
(ii) P is unable is row when someone else is in the boat. 5. Two wattmeter method is used for measurement of power in
(iii) Q is unable to row with anyone else except R. a balanced three-phase load supplied from a balanced three-
(iv) Each person rowed for at least one trip. phase system. If one of the wattmeters reads half of the
(v) Only one person can row during a trip. other (both positive), then the power factor of the load is
Who rowed twice? (a) 0.532 (b) 0.632
(a) P (b) Q (c) 0.707 (d) 0.866
(c) R (d) S 6. Consider a lossy transmission line with V1 and V2 as the
sending and receiving end voltages, respectively. Z and X
Technical Section Test are the series impedance and reactance of the line,
QUESTIONS 1 – 25 CARRY TWO MARKS EACH. respectively. The steady-state stability limit for the
transmission line will be
1. A single-phase 100 kVA, 1000 V/100 V, 50 Hz transformer has
a voltage drop of 5% across its series impedance at full load. V1V2 V1V2
Of this, 3% is due to resistance. The percentage regulation (a) greater than X
(b) less than
X
of the transformer at full load with 0.8 lagging power factor is
V1V2 V1V2
(a) 4.8 (b) 6.8 (c) equal to (d) equal to
X Z
(c) 8.8 (d) 10.8
7. The graph of a network has 8 nodes and 5 independent loops.
2. In a salient pole synchronous motor, the developed reluctance The number of branches of the graph is
torque attains the maximum value when the load angle in (a) 11 (b) 12
electrical degrees is
(c) 13 (d) 14
(a) 0 (b) 45 8. In the figure, the voltages are v1(t) = 100 cos(wt), v2(t) =
(c) 60 (d) 90
æ pö æ pö
3. A single-phase fully controlled rectifier is supplying a load 100cos çè wt + ÷ø and v3(t) = 100cos çè wt + ÷ø . The circuit
with an anti-parallel diode as shown in the figure. All switches 18 36
and diodes are ideal. Which one of the following is true for is in sinusoidal steady-state, and R < < wL. P1, P2 and P3 are
instantaneous load voltage and current? the average power outputs. Which one of the following
statements is true?
4
R L L R (a) Y = ABCD (b) Y = (A + B) (C + D)
(c) Y = A + B + C + D (d) Y = AB + CD
15. The op-amp shown in the figure is ideal. The input impedance
P1 P2 P3 Vin/iin is given by
+ + + Z
v1(t) v2(t) v3(t)
– – – Z
iin
(a) P1 = P2 = P3 = 0 (b) P1 < 0, P2 > 0, P3 > 0 +
(c) P1 < 0, P2 > 0, P3 < 0 (d) P1 > 0, P2 < 0, P3 > 0 V0
9. Match the transfer functions of the second-order systems –
with the nature of the system given below. Vin +

Transfer functions Nature of system
15
P. I. Overdamped R1
2 R2
s + 5s + 15
25
Q. 2 II. Critically damped
s + 10s + 25 R1 R2
35 (a) Z
R2 (b) - Z R
1
R. 2 III. Underdamped
s + 18s + 35 R2
(a) P-I, Q-II, R-III (b) P-II, Q-I, R-III (c) Z (d) - Z R + R
(c) P-III, Q-II, R-I (d) P-III, Q-I, R-II 1 2
10. A positive charge of 1 nC is placed at (0, 0, 0.2) where all 16. A continuous-time input signal x(t) is an eigen function of an
dimensions are in meters. Consider the x-y plane to be a LTI system, if the output is
conducting ground plane. Take Î0 = 8.85 × 10–12 F/m. The Z (a) k x(t), where k is an eigenvalue.
component of the E field at (0, 0, 0.1) is closed to (b) kejwt x(t), where k is an eigenvalue and ejwt is a complex
(a) 899.18 V/m (b) –899.18 V/m exponential signal.
(c) 999.09 V/m (d) –999.09 V/m (c) x(t) ejwt. where ejwt is a complex exponential signal.
11. Let f be a real-valued function of a real variable defined as (d) k H(w), where k is an eigenvalue and H(w) is frequency
f(x) = x2 for x ³ 0, and f(x) = –x2 for x < 0. Which one of the response of the system.
following statements is true? 17. Consider a non-singular 2 × 2 square matrix A. If trace (a) = 4
(a) f(x) is discontinuous at x = 0. and trace (A2 ) = 5, the determinant of the matrix A is
(b) f(x) is continuous but not differentiable at x = 0. _________ (up to 1 decimal place).
(c) f(x) is differentiable but its first derivative is not 18. Let f be a real valued function of a real variable defined as f(x)
continuous at x = 0. = x –[x], where [x] denotes the largest integer less than or
(d) f(x) is differentiable but its first derivative is not
differentiable at x = 0. 1.25
12. The value of the directional derivative of the function f(x, y,
z) = xy2 + yz2 + zx2 at the point (2, –1, 1) in the direction of the
equal to x. The value of ò f ( x ) dx ________ (up to 2
0.25
vector p = i + 2j + 2k is
(a) 1 (b) 0.95 decimal places).
(c) 0.93 (d) 0.9 19. In the two-port network shown, the h 11 parameter
z +1
13. The value of the integral Ñò z2 - 4 dz in counter clockwise æ V1 ö
çè where, h11 = I , when V2 = 0÷ø in ohms is _________
C i
direction around a circle C of radius 1 with center at the point (up to 2 decimal places).
z = –2 is
pi 2I1
(a) (b) 2pi
2
pi 1W 1W
(c) - (d) –2pi
2
14. In the logic circuit shown in the figure, Y is given by I1
A V1 1W V2
B
Y
C
D
5
20. The series impedance matrix of a short three-phase
mVr 2 N p2 w mIr 2 N P NSw
é Zs Zm Zm ù (a) (b)
ê ú I V
transmission line in phase coordinates is ê Zm Zs Zm ú .
êë Zm Zm Zs úû mVr 2 N 2p w mIr 2 N 2P w
(c) (d)
If the positive sequence impedance is (1 + j10) W, and the 2I 2V
zero sequence is (4 + j31) W, then the imaginary part of Zm (in 27. A 0-1 ampere moving iron ammeter has an internal resistance
W) is ___________ (up to 2 decimal places). of 50 mW and inductance of 0.1 mH. A shunt coil is connected
to extend its range to 0-10 Ampere for all operating
21. The positive, negative and zero sequence impedances of a frequencies. The time constant in milliseconds and resistance
125 MVA, three-phase, 15.5 kV, star-grounded, 50 Hz in mW of the shunt coil respectively are
generator are j0.1 pu, j0.05 and j0.01 pu respectively on the (a) 2, 5.55 (b) 2, 1
machine rating base. The machine is unloaded and working (c) 2.18, 0.55 (d) 11.1, 2
at the rated terminal voltage. If the grounding impedance of 28. The positive, negative and zero sequence impedances of a
the generator is j0.01 pu, then the magnitude of fault current three-phase generator are Z1, Z2 and Z0 respectively. For a
for a b-phase to ground fault (in kA) is _________ (up to 2 line-to-line fault with fault impedance Zf, the fault current is
decimal places). If1 = kIf, where If is the fault current with zero fault impedance.
22. A 1000 × 1000 bus admittance matrix for an electric power The relation between Zf and k is
system has 8000 non-zero elements. The minimum number of
branches (transmission lines and transformers) in this system Zf =
( Z1 + Z2 ) (1 - k ) ( Z1 + Z2 ) (1 + k )
(a) (b) Zf =
are_____(up to 2 decimal places). k k
23. The waveform of the current drawn by a semi-converted from
Zf =
( Z1 + Z2 ) k ( Z1 + Z2 ) k
a sinusoidal AC voltage source is shown in the figure. If I0 = 20 (c) (d) Zf =
A, the rms value of fundamental component of the current is 1- k 1+ k
________ A (up to 2 decimal places). 29. Consider the two bus power system network with given loads
as shown in the figure. All the values shown in the figure are
Voltage and
curent in per unit. The reactive power supplied by generator G1 and
Vm sin (w t) G2 are QG1 and QG2 respectively. The per unit values of QG1,
QG2 and line reactive power loss (Qloss) respectively are
I0 1.0Ðd 1.0Ð0°
wt j0.1
0 I0
30° 180° G1 G2
Qloss
210° 20 + jQG1 15 + jQG2
24. A separately excited dc motor has an armature resistance Ra
= 0.05 W. The field excitation is kept constant. At an armature 15 + j5 20 + j10
voltage of 100 V, the motor produces a torque of 500 Nm at (a) 5.00, 12.68, 2.68 (b) 6.34, 10.00, 1.34
zero speed. Neglecting all mechanical losses, the no-load (c) 6.34, 11.34, 2.68 (d) 5.00, 11.34, 1.34
speed of the motor (in radian/s) for an armature voltage of 30. The per unit power output of a salient pole generator which
150 V is _________ (upto 2 decimal places). is connected to an infinite bus, is given by the expression, P
25. Consider a unity feedback system with forward transfer = 1.4 sind + 0.15 sin2d, where d is the load angle. Newton-
function given by Raphson method is used to calculate the value of d for P = 0.8
pu. If the initial guess is 30°, then its value (in degree) at the
1 end of the first iteration is
G ( s) =
( s + 1)( s + 2) (a) 15° (b) 28.48°
(c) 28.74° (d) 31.20°
The steady-state error in the output of the system for a unit- 31. A DC voltage source is connected to a series L-C circuit by
step input is ________ (up to 2 decimal places). turning on the switch S at time t = 0 as shown in the figure.
QUESTIONS 26 – 55 CARRY TWO MARKS EACH. Assume i(0) = 0, v(0) = 0. Which one of the following circular
loci represents the plot of i(t) versus v(t)?
26. A transformer with toroidal core of permeability m is shown
in the figure. Assuming uniform flux density across the S i(t)
circular core cross-section of radius r < < R, and neglecting
any leakage flux, the best estimate for the mean radius R is
t=0 L=1H
r iS = 0
+ iP = I sinwt +
+ +
5V C = 1 F v(t)
NP NS –
R VS –
VP = V coswt
– –
6
i(t) i(t)

v(t)
D Q D Q D Q
(0, –5) fOUT
(a) (b) v(t)
(5, 0) C C C

fIN
i(t) i(t) (a) It can be used for dividing the input frequency by 3.
(b) It can be used for dividing input frequency by 5.
(c) It can be used for dividing the input frequency by 7.
(c) (d) v (t)
(d) It cannot be reliably used as a frequency divider due to
(0, 5)
(–5, 0) disjoint internal cycles.
37. Digital input signals A, B, C with A as the MSB and C as the
v (t) LSB are used to realize the Boolean function
F = m0 + m2 + m3 + m5 + m7, where mi denotes the ith minterm.
32. The equivalent impedance Zeq for the infinite ladder circuit In addition, F has a don’t care for m1 . The simplified
shown in the figure is expression for F is given by
j9W j9W (a) AC + BC + AC (b) A + C
(c)
C+A (d) AC + BC + AC
j5 W j5 W 38. Consider the two continuous-time signals defined below:
Zeq
ì t , -1 £ t £ 1ü ì1 - t , - 1 £ t £ 1 ü
–1j1 W 1j1 W x1 (t ) = í ý , x2 (t ) = í ý
î0, otherwiseþ î0 , otherwiseþ
These signals are sampled with a sampling period of
(a) j12 W (b) –j12 W T = 0.25 seconds to obtain descrete-time signals x1[n] and
(c) j13 W (d) 13 W x2[n], respectively. Which one of the following statements is
33. Consider a system governed by the following equations: true?
dx1 ( t ) (a) The energy of x1[n] is greater than the energy of x2[n].
= x 2 ( t ) - x1 ( t )
dt (b) The energy of x2[n] is greater than the energy of x1[n].
dx 2 ( t ) (c) x1[n] and x2[n] have equal energies.
= x1 ( t ) - x 2 ( t )
dt (d) Neither x1[n] nor x2[n] is a finite energy signal.
The initial conditions are such that x 1 (0) < x 2 (0) 39. The signal energy of the continuous-time signal
< ¥. Let x1f = L im x1 ( t ) and x 2f = L im x 2 ( t ) . Which x(t) = [(t – 1) u(t – 1)] – [(t – 2) u(t – 2)] – [(t – 3)
t ®¥ t ®¥ u(t – 3)] + [(t – 4) u(t – 4)] is
one of the following is true?
(a) x1f < x2f < ¥ (b) x2f < x1f < ¥ 11 7
(a) (b)
(c) x1f = x2f < ¥ (d) x1f = x2f = ¥ 3 3
34. The number of roots of polynomial, s7 + s6 + 7s5 + 14s4 +
31s3 + 73s2 + 25s + 200, in the open left half of the complex 1 5
(c) (d)
plane is 3 3
(a) 3 (b) 4 40. The Fourier transform of a continuous time signal x(t) is given
(c) 5 (d) 6
1
z2 X (w) = . - ¥ < w < ¥,
by where j = -1 and w
f (z) = , (10 + jw )2
(z )
35. If C is a circle |z| = 4 and 2 2 then
- 3z + 2 denotes frequency. Then the value of | l nx(t)| at t = 1 is
_______ (upto 1 decimal place). (ln denotes the logarithm
Ñò f ( z) dz is to base e).
C 41. In the circuit shown in the figure, the bipolar junction
(a) 1 (b) 0 transistor (BJT) has a current gain b = 100. The base-emitter
(c) –1 (d) –2 voltage drop is a constant, VBE = 0.7 V. The value of the
36. Which one of the following statements is true about the Thevenin equivalent resistance RTh (in W) as shown in the
digital circuit shown in the figure? figure is _________ (upto 2 decimal places).
7
a 47. A three-phase load is connected to a three-phase balanced
supply as shown in the figure.
10 kW
If Van = 100Ð0° V, Vbn = 100Ж120°V and Vcn = 100Ð – 240°V
+ (angles are considered positive in the anti-clockwise
10 kW RTh
15 V 1 kW direction), the value of R for zero current in the neutral wire is
+ ________ W (upto 2 decimal places).

10.7 V a

b R
42. As shown in the figure, C is the arc from the
point (3, 0) to the point (0, 3) on the circle x2 + y2 = 9. The n
value of the integral ò ( y2 + 2yx ) dx + (2xy + x 2 ) dy is
–j10 j10
C c
_________ (upto 2 decimal places).
y
b
(0, 3)
C 48. The voltage across the circuit in the figure and the current
through it, are given by the following expressions:
v(t) = 5 – 10 cos(wt + 60°) V
i(t) = 5 + X cos(wt) A
x where, w = 100 p radian/s. If the average power delivered to
(3,0) the circuit is zero, then the value of X(in ampere) is
_________ (upto 2 decimal places).
43. Let f(x) = 3x3 – 7x2 + 5x + 6. The maximum value of f(x) over
the interval [0, 2] is ________ (upto 1 decimal place). i(t)
+
é1 0 - 1 ù
ê ú
44. Let A = ê -1 2 0ú and B = A3 – A2 – 4 A + 5I, where I is
êë0 0 - 2 úû v(t) Electrical
the 3 × 3 identify matrix. The determinant of B is _________ circuit
(upto 1 decimal place).
45. The capacitance of an air-filled parallel-plate capacitor is 60

pF. When a dielectric slab whose thickness is half the
distance between the plates, is placed on one of the plates
covering it entirely, the capacitance becomes 86 pF. 49. A phase controlled single-phase rectifier, supplied by an AC
Neglecting the fringing effects, the relative permittivity of source, feeds power to an R-L-E load as shown in the figure.
the dielectric is __________ (upto 2 decimal places). The rectifier output voltage has an average value given by
Vm
46. The unit step response y(t) of a unity feedback V0 = ( 3 + cos a ) , where Vm = 80p volts and a is the
system with open-loop transfer function (s) H (s) 2p
K firing angle. If the power delivered to the lossless battery is
= is shown in the figure. 1600 W. a is degree is ________ (upto 2 decimal places).
(s + 1) (s + 2)
2

+
The value of K is _________ (upto 2 decimal places).

y(t)
2W
1.4
1.2
Vm sin(wt ) V0
1
10 mH
0.8
0.6 +
0.4 80 V

0.2 – Battery
0
0 2 4 6 8 10 12 14 16 18 20 t(sec)
8
50. The figure shows two buck converters connected in parallel. D
iL L = 5 mH
The common input dc voltage for the converters has a value
of 100 V. The converters have inductors of identical value.
The load resistance is 1 W. The capacitor voltage has
negligible ripple. Both converters operate in the continuous +
50 V B1 S B2 150 V
conduction mode. The switching frequency is 1 kHz, and the –
switch control
signals are as shown. The circuit operates in the steady-
state. Assuming that the converters share the load equally,
the average value of is1, the current of switch S1 (in ampere)
is ________ (upto 2 decimal places). 54. The equivalent circuit of a single-phase induction motor is
shown in the figure, where the parameters are
is1
R1 = R '2 = X l1 = X 'l 2 = 12W, X M = 240W and s is the
S1 slip. At no-load the motor speed can be approximated to be
+ L the synchronous speed. The no-load lagging power factor
100 V C 1W of the motor is ________ (up to 3 decimal places).

R1 jX l1

S2 L

2s
XM
j X j2¢
Switch control signals 2 j
2
S1 V Ð 0°
t R2¢
XM 2( 2 - s)
S2 j
2 X j2¢
t j
0 0.5 ms 1 ms 2

51. A three-phase, 900 kVA, 3 kV/ 3kV ( D / Y ) , 50 Hz


transformer has primary (high voltage side) resistance per
phase of 0.3 W and secondary (low voltage side) resistance 55. The voltage v(t) across the terminals a and b shown in the
per phase of 0.02 W. Iron loss of the transformer is 10 kW. figure, is a sinusoidal voltage having a frequency w = 100
The full load % efficiency of the transformer operated at radian/s. When the inductor current i(t) is in phase with the
unity power factor is ________ (upto 2 decimal places). voltage v(t), magnitude of the impedance
52. A 200 V DC series motor, when operating from rated voltage Z(in W ) seen between the terminals a and b is ________
while driving a certain load, draws 10 A current and runs at (upto 2 decimal places).
1000 rpm. The total series resistance is 1 W. The magnetic
circuit is assumed to be linear. At the same supply voltage, i(t)
the load torque is increased by 44%. The speed of the motor V(t) +
in rpm (rounded to the nearest integer) is ___________. a L
53. A dc to dc converter shown in the figure is charging a battery
bank, B2 whose voltage is constant at 150 V. B1 is another Z
battery bank whose voltage is constant at 50 V. The value of 100 W
the inductor, L is 5 mH and the ideal switch, S is operated 100 µ F
with a switching frequency of 5 kHz with a duty ratio of 0.4.
Once the circuit has attained steady state and assuming the
diode D to be ideal, the power transferred from B1 to B2 (in –
Watt) is _________ (up to 2 decimal places). b
9

General Aptitude Test


1. (a) Course, Coarse 9. (c) Suppose total marbles = 100
2. (d) Convoluted and cost of each marble = Rs. 1
3. (d) \ Three roots of f (x) = 0 are –2, 0, 3 Then, original cost of all marbles = Rs. 100
As, new cost of all marbles = Rs. 125
Then, f (x) = (x + 2) (x) (x–3)
Þ f (x–3) = (x–1) (x–3) (x–6) 100
Therefore, 1 Rs. will be the cost of marbles
So, f (x–3) = 0 x = (1, 3, 6) 125
100 ´ 100
(k + 2)2 Hence, 100 Rs. will be cost of i.e. 80 marbles
4. (c) \ Given expression is 125
k -3 10. (d) In first forward trip, Q and R will go while Q will row.
Using given options In first return trip, R will came back and also R will row.
If k = 4, In second forward trip, P and R will go while R will
Row. In second return trip, P will come back and also
(k + 2)2 (4 + 2) 2 P will Row.
= = 36 (an integer)
k -3 4-3 In third forward trip P and S will go while S will Row.
So, R Rowed twice.
(k + 2)2 102
If k = 8, = = 20 (an integer)
k -3 8-3 Technical Section Test
1. (a) As given that,
(k + 2)2 (30)2 The full load voltage drop due to P.u. impedance
when k = 28, = = 36 (an integer)
k -3 25 5
Hence option (c) is correct ( Z Pu ) = = 0.05 Pu
100
5. (a) \ F (a,b) = (a–b)2 The full load voltage drop due to P.u. resistance
\ F (1, 3) = (1– 3)2 = 4 3
As, G (a,b) = êa–b êso G (1 , 3) = ê1 – 3 ê = 2 ( RPu ) = = 0.03 Pu
100
Hence, G ( F (1,3) , G (1, 3) ) = G (4 , 2) = ê4 – 2 ê = 2 We know that,
6. (c) Here, total digits = 10. As, total alphabets = 26 The Reactance,
Hence req. no. of distinct passwords X Pu = 2
(Z Pu 2
- RPu )
= 10 C1 ×26 C1 ×26 C1 × 3! = 40,560
= (0.05) 2 - (0.03) 2 = 0.04 Pu
A M C C N K U Now, the percentage regulation of the transformer at
7. (c) Here, +4 +4 +4 +4 +4 +4 +4 full load with 0.8 lagging power factor is
E Q G J R O Y V .R. = ( RPu cos f + X Pu sin f) ´100
= (0.03 ´ (0.8) + 0.04 ´ (0.6)) ´100
D H L P
Hence, +4 +4 +4 +4 V .R. = 4.8%
H L P T 2. (b) As we know that,
Salient Pole synchronous motor torque relation per
8. (b) \ N(B) = N(G) + 2 and N(B) + N(G) = 12 Þ N(B) =
phase is :
7, N(G) = 5
No. of ways of picking 3 children = 12C3 1éV 2 é 1 1 ù ù
Favourable no. of ways (No. of girls & No. of Boys) trel = ê t ê - ú sin 2d ú
Wmêë 2 êë X q X d úû úû
= (5C2 × 7C1) + 5C3 = (10 × 7) + 10 = 80
In this equation second term is reluctance torque,
80 8 which is directly propertional to sin(2d).
Hence, required probability = 12
=
220
C3 So, reluctance torque will be maximum, when, (d = 45°)

325 Þ sin 2d = sin 90°


= 0.36 = sin(2 ´ 45°) Þ sin 90° = 1 (Maximum).
864
10
3. (c) As given circuit :- Þ TR1AC is a combination of two anti-parallel
thyristors. So, when current flows from (MT1 to MT2)
i0 and MT1 is positive and MT2 is negative.
i0 + TR1AC :-
D1 D3 L + –
O G
MT1 MT1
A V0
D
MT2
D2 D4 – MT2 +
G –
(TR1AC) (TR1AC)

Let us consider, D1 is ON, D2 is off, D3 is off and D4 One of the thyristor conduct but not the other.
is ON, this is forward bias condition. Similarly when MT1 is negative and MT2 is positive
then current flows from MT2 to MT1 then second of
V1n (t ) = m m sin(wt ), the thyristor conducts but not the first one, so, given
V0 (t ) = mn sin(wt ) direction of current is possible.
Similarly if D1 is off, D2 is ON, D3 is ON, and D4 is GTO also conduct only in the direction of A to K. So
off, D4 is off, this is a reverse bias condition. given direction is not possible.

V0 (t ) = Vm sin(wt ) (Positive) A
G K
V1n ()t Conduction
I
K
Vm
In MOSFET, an inherent diode is present as shown
in figure :-
wt
0 p 2p D D
–Vm
V0
v0 ³ 0 S
Vm G
S

wt Case I : When S is +ve and D is -ve then dioide is


a p p+a 2p
(for continuous
ON.
i0 > 0 conductions)
i0 D

I
wt
i0 i0 > 0 (for discontinuous
conductions) S
I0 I0
wt
Case II : When S is -ve and D is +ve then diode is
So, V0 ³ 0, i0 ³ 0 for any instant, current only in one off.
direction.
4. (b) Thyristor conductor only in the direction of A to K, D (+ve)
if properly triggered, so conduction from (K to A) is
not possible.
I S (–ve)
Thyristor:-
Hence, flow of current is possible.
A 5. (0.866)
I As we know that,
G In two waltmeter of power measurement
11
é 3(W1 - W2 ) ù éV V ù
tan(f) = ê ú Pmax 2 = ê 1 2 (sin d )ú
êë (W1 + W2 ) úû ë X û

éV V ù
é 3(W1 - W2 ) ù
-1
Pmax 2 = ê 1 2 (sin 90°)ú (Q d = 90°)
f = tan ê ú ë X û
ëê (W1 + W2 ) ûú
éV V ù
Pmax 2 = ê 1 2 ú
æ W ö ë X û
As given that, Q çW2 = 1 ÷
è 2 ø
Hence, Pmax2 > Pmax1
é æ W öù \ Steady state stability limit for transmission line
ê 3 ç W1 - 1 ÷ ú
è 2 ø V1V2
f = tan -1 ê ú Pmax 2 = .
ê æ W1 ö ú X
ê W +
ç 1 2 ÷ ú
ë è ø û 7. (b) As given that,
the number of nodes (n) = 8
the number of loops (l) = 5
é W ù
ê 3´ 1 ú Number of branches = ?
f = tan -1 ê 2
3W ú As, we know that, in a network,
ê 1 ú l = b – (n – 1)
ëê 2 ûú 5 = b – (8 – 1)
5=b–7
é 1 ù
f = tan -1 ê ú = 30° b = 12
ë 3û
8. (c) As given that,
f = 30°
V1 (t ) = 100cos(wt ) = 100 cos(wt + 0°)
So, the power factor = cos f = cos30°
æ pö
æ 3ö V2 (t ) = 100 cos ç wt + ÷ = 100 cos(wt + 10°)
=ç = 0.866. è 18 ø
ç 2 ÷÷
è ø æ pö
6. (b) Using short line model : V3 (t ) = 100cos ç wt + ÷ = 100cos(wt + 5°)
è 36 ø
éA Bù é1 2 ù é 1 R + jX ù V2
êC D ú = ê0 1 ú = ê0 1 úû
ë û ë û ë
Let, we assume, Z = | Z | Ðq°.
\ The receiving end power with impudence Z is :
V3
V1 V2
Z
10°
V1V2 V2
P= cos(q - d ) - 2 cos q 5°
|Z| |Z|
V1
For, fix value of V1, V2 and Z, (P) max1 will occur for
d = q. So, | V1 (t ) |=| V2 (t ) |=| V3 (t ) |= 100
éV V V2 ù ÐV1 (t ) = 0°, ÐV2 (t ) = 10°, ÐV3 (t ) = 5°
Pmax1 = ê 1 2 cos(0) - 2 cos q ú
ë|Z | |Z| û Thus V2 leads V1 and V3.
So, V2 is a source, V1 and V3 are absorbing.
éV V V2 ù Hence, P2 > 0, P1 , P3 < 0
Pmax1 = ê 1 2 - 2 cos(d )ú (Q q = d)
ë|Z | |Z | û 9. (c) As given that,
Now, receiving end power with reactance X, is : é 15 ù
jx (i) Transfer function (r) = ê 2 ú
V1 V2 ë (s + 5s + 15) û
By comparing with standard transfer function.
12
Total electric field at point (P) due to charge Q is :
é Wn2 ù r
P=ê 2 2ú r QR12
ë s + 2xWn S + Wn û E12 = r
4p Î0 | R12 |3
2xWn = 5, Wn2 = 15
r 1´10-9 (-0.1aˆ z ) -(1´10-9 )(0.3aˆ z )
Wn = 15 = 3.872 rad/sec (underdamped) E12 = +
4p(8.854 ´10-12 )(0.1)3 4p(8.854 ´ 10-12 ) ´ (0.3)3
So, 2x ´ (3.872) = 5
é -105 105 ù
=ê - ú aˆ z
5
x= = 0.64. ë 4p(8.854) 4p(8.854 ´ 9) û
(2 ´ 3.872)
= [-898.774 - 99.863]aˆ z
é 25 ù @ [-999.09aˆ z ] V / m
(ii) Given Transfer function Q = ê 2 ú
ë s + 10s + 25 û 11. (d) As given that,
By comparing this with standard transfer function.
f ( x) = x2 for x ³ 0
é Wn2 ù
Q=ê 2 2ú f ( x) = - x2 for x < 0
ë s + 2xWn s + Wn û
So,
Wn2 = 25, 2xWn = 10 ìï- x 2 , x < 0 üï
f ( x) = í ý
Wn = 5, 2x ´ 5 = 10 2
ïî x , x ³ 0 ïþ
f(0) = 0
x=1 (Critically damped)
So, f(x) is continuous and differentiable at x = 0,
(iii) Given, transfer function,
ì -2 x, ( x < 0) ü
é 35 ù f '( x ) = í ý
R=ê 2 ú î 2 x, ( x ³ 0) þ
ë s + 18s + 35 û
f '(0) = 0.
é Wn2 ù So, f(x) is differentiable at (x = 0).
=ê 2 2ú
ë s + 2xWn s + Wn û ì 2, x < 0 ü
f ''( x) = í ý
î -2, x ³ 0 þ
Wn2 = 35, 2xWn = 18 Þ xWn = 9
f ''(0) = 2, –2
9 The first derivation f '(x) is not differentiable at x =
Þx= >1 (over damped) 0. Hence, f(x) is differentiable but its first derivative
35
is not differentiable at x = 0.
Hence, (P = III, Q = II and R = I)
12. (a) As given that directional derviatives :-
Option (c) is correct.
10. (d) f( x, y, z ) = xy 2 + yz 2 + zx 2
So,
æ df ö ˆj çæ df ÷ö + kˆ æ df ö
Z
(Ñf) = iˆ ç ÷ + ç ÷
è dx ø è dy ø è dz ø
(0, 0, 0.2) (Q = 1 nc) = [iˆ( y 2 + 2 zx ) + ˆj(2 xy + z 2 ) + kˆ(2 yz + x 2 )]

P (0, 0, 0.1) Ñf(2, - 1, 1) = iˆ(1 + 4) + ˆj (-4 + 1) + kˆ(-2 + 4)


0.4
(z = 0)
Ñf(2, -1, 1) = [5iˆ - 3 ˆj + 2 kˆ]
y So, required directional derivative at (2, –1, 1) in the
P' (0, 0, –0.1)
direction of Pˆ (iˆ + 2 ˆj + 2kˆ) is
(Q = –1 nc)
X (Ñf) Pˆ é (iˆ + 2 ˆj + 2kˆ) ù
(0, 0, –0.2) (Ñf) at P = = [5iˆ - 3 ˆj + 2kˆ] ´ ê ú
| Pˆ | êë 1 + 4 + 4 úû

(5 - 6 + 4)
= = 1.
3
13
13. (a) According to virtual ground,
Y
VA = VB = Vin = V
At node A,
éV - V0 ù
iin = ê in ú ...(i)
C ë Z û
X According to voltage divider principle
–4 –3 –2 –1 2 3 4
é V ´ R2 ù
Vin = ê 0 ú
ë R1 + R2 û
Vin ( R1 + R2 )
By Cauchy's integral formula or V0 = R2
æ z +1 ö By putting the value of V0 in equation (i)
ÞÑ
ò çè z 2 - 4 ø÷dz
C é éVin ( R1 + R2 ) ù ù
êVin - ê úú
( z + 1) ë R2 ûú
ÞÑ iin = ê
ò ( z - 2)( z + 2)
dz ê Z ú
C ê ú
ëê ûú
é ( z + 1) ù
ê ( z - 2) ú æ V ö é R - ( R1 + R2 ) ù -Vin æR ö
Þ Ñò ê ú dz Z = ç in ÷ ê 2 ú= ´ç 1 ÷
C
ê ( z + 2) ú è iin ø ë R2 û iin è R2 ø
ê ú
ë û æ R ö æ - ZR2 ö
Vin
= (- Z ) ´ ç 2 ÷ = ç
é æ z + 1 öù iin è R1 ø è R1 ÷ø
= 2 pif ( -2) êQ f ( z ) = ç z - 2 ÷ ú
ë è øû 16. (a) Let us consider an eigen function is:
X(t) = est
é -2 + 1 ù é -1 ù æ pi ö
= 2pi ê ú = 2pi ê ú = ç ÷ .
ë -2 - 2 û ë -4 û è 2 ø LTI System Y(t) = h(t) × x(t)
14. (d) As given circuit is : X(t) or
h(t) st
Y(t) = H(s) . e
A AB
AB where, H(S) = Laplace transform of h(t).
B
NAND X(t) = System input or eigen function
Y Y
H(t) = Tansfer function of system
C NAND CD Y(t) = System output.
D CD For a signal to be eigen function of an L.T.I. System.
NAND Thus, Y(t) = Kx (t) = K . eSt
where Kis eigen value and X(t) is eigen function input.
Output, Y = ( AB CD ) = ( AB + CD )
17. (5.5) Let l1 and l2 are the eigen value of A, then trace (A)
Q By Demorgan's theorem
Þ (l1 + l2) = 4 and l12 and l22 will be eigen value
Y = AB + CD
of (A2) then (l12 + l 22 = 5) .
Z We know that,
15. (b)
iin (l1 + l 2 ) 2 = (l12 + l 22 ) + 2l1l 2
+ Þ 16 = 5 + 2l1l 2
+ V0
Vin – Þ 2l1l 2 = 11

V 11
R1 Þ l1l 2 = = 5.5
2
R2 Q| A | is the product of eigen value.

So, | A |= 5.5
14
1.25 1.25 V2 - 1( I 2 + 2 I1 ) - 1( I1 + I 2 ) = 0
18. (0.5) I =
ò f ( x) dx = ò | x - [ x] | dx
2 I 2 + 3I1 = 0 Þ 2 I 2 = -3I1
0.25 0.25
-3
1 1.25 Þ I2 = I1 ...(ii)
= ò ( x - | x |) dx + ò [ x - [ x]] dx 2
By putting the value of I2 from equation (ii) to
0.25 1
equation (i).
1 1.25
3 ö æ 4 I - 3I1 ö I1
= ò x dx + ò ( x - 1) dx æ
V1 = ç 2I1 - I1 ÷ = ç 1 ÷=
0.25 1 è 2 ø è 2 ø 2
1 1.25
é x2 ù é x2 ù V1 1
=ê ú + ê - xú Þ =
I1 2
ë 2 û 0.25 ë 2 û1
By using two port N/W :-
é 1 (0.25) ù é é (1.25) 2ù é 1 ùù 2
V1 1
=ê - ú + êê - 1.25ú - ê - 1ú ú h11 = = = 0.5 W
ë2 2 û êë ë 2 û ë 2 û úû I1 2
= 0.5 20. (7) As given that,
19. (0.5)As given two port Network, The positive sequence impedance,
Z1 = (1 + j10) W
The zero sequence impedence,

1W 1W Z0 = (4 + j31) W
+ + As we know that
I1 Z1 = ( Z s - Z m ) = (1 + j10) ...(i)
1W
V1 V2 Z 0 = ( Z s + 2Z m ) = (4 + j31) ...(ii)
By subtracting equation from (ii) to (i)
– – ( Z0 - Z1 ) = (4 + j31 - 1 - j10)
Let, V2 = 0, then the terminals of Port 2 will be shot Þ Z 0 - Z1 = (3 + 21 j )
circuited.
Þ Z0 - Z1 = 3(1 + 7 j )
1W Va 1W ( Z s + 2Z m ) - (Z s - Z m ) = 3(1 + 7 j)
I1 Þ 3Z m = 3(1 + 7 j )
I1
Þ Z m = (1 + 7 j )
1W
Hence, imaginary part of Zm is 7.00.
V2=0
21. (73.52) As given that,
– Positive sequence impedence (Z1) = j 0.1 Pu
Negative sequence impedence (Z2) = j 0.05 Pu
Zero sequence impedence (Z0) = j 0.01 Pu
By using KCL at V0. Grounding impedance (Zn) = j 0.01 PU
Va Va Va - V1 As the machine is unloaded and operating at rated
+ + + 2 I1 = 0 voltage, so, Vth = 1 P.u.
1 1 1
For L.G. Fault :-
3Va - V1 + 2I1 = 0 The fault current for phase to ground fault,
æ V1 - Va ö é 3 Vth ù
Again, ç ÷ = I1 I fPu = ê ú
è -1 ø ë Z1 + Z 2 + Z 0 + 3Z n û
By using KVL in first loop, we get,
é 3 ´1 ù
V1 - 1I1 - ( I1 + I 2 ) = 0 =ê ú
ë 0.1 + 0.05 + 0.01 + 3 ´ 0.01 û
V1 = 2 I1 + I 2 ...(i)
Now, by using KLV in second loop, we get 3
= ( Pu ) = 15.789 Pu
0.19
15
Now, base current
2 2 ´ 20 æ 30° ö
= cos× ç
é r ù é 125 ´106 ù p è 2 ÷ø
(Ib ) = ê ú= ê 3ú
= 4656.05 A
êë 3 × V2 úû ë 3 ´15.78 ´10 û
2 2 ´ 20
= (cos15°)
I f (actual) = ( I f ) Pu ´ ( Ib )Base p
= 15.789 ´ 4656.05 = 73516.538
2 2
= 73.52 K.Amp. = ´ 20 ´ (0.9659)
22. (3500) p
As given that, ( I st )rms = 17.392 Amp.
Total number of elements in YBus 24. (600)
(Admittance matrix) = 1000 ´ 1000 = 106 As given that,
Number of diagonal element (N) = 1000 Armature resistance (Ra) = 0.05 W
Power system has non-zero elements are 8000. Torque produced by motor = 500 N-m
So, total number of element, which are non-zero as Field excitation is kept constant, so f is constant.
well as non diagonal Armature voltage (Vt) = 100 V.
= (8000 - 1000) = 7000. As we know that,
\ Total number of branches at zero speed, such that
é (Number of non-zero off diagonal element) ù ( Eb ) = K fW = 0
=ê ú
ë 2 û So, Armature current
7000 éV - Eb ù é100 - 0 ù
= = 3500. (I a ) = ê ú =ê
2 ú = 2000 Amp
ë Ra û ë 0.05 û
23. (17.39)
And Torque (T ) = K fI a
Voltage
é T ù é 500 ù 5 1
Kf = ê ú = ê ú= = .
ë I a û ë 2000 û 20 4
Current When motor runs on no-load given all mechanical
(I)0
losses neglected. No load current is negligible and
the voltage drop at no. load can be neglegible.
wt
O Eb ; V = 150 V
\ Eb = K fWm
Eb é150 ´ 4 ù
Wm = = = 600 rad/sec.
K f êë 1 úû
25. (0.66)
As given that,
I0
I0
Forward transfer function with (unity feedback)
p+a 2T q = wt system
(a = 30°) –I0
–I0 1
G(S ) =
210° (S + 1)( S + 2)
We know that, for unit step input when H(S) = 1,
As given that, I0 = 20 Amp, a = 30°
As we know that, \ K p = lim[G ( S ) × H ( S )]
S ®0
¥
é 4I0 ù é na ù é na ù é ù æ1ö
i0 (t ) = å ê np ú × cos ê 2 ú × sin ê nwt - 2 ú
ë û ë û
= lim ê
1
ú =ç ÷
n =1, 3, 5 ë û S ®0 ( S + 1)( S + 2)
ë û è2ø
So, R.M.S. value of fundamental current (Ist) is : For unit step input, steady state error for type - 0 and
step input.
æ 2 2 I0 ö æaö
( I st )rms = ç ÷÷ cos ç ÷ é 1 ù
ç p è2ø
è ø ess = ê ú
ëê1 + K p ûú
16
To make the meter indendent, all frequencies, time
é ù constants of the both shunt and meter arm should be
ê 1 ú 1´ 2 2
=ê ú= = = 0.66 units equal.
ê æ 1 öú 3 3
1 +
ê çè 2 ÷ø ú WLm WLsh
ë û =
Rm Rsh
26. (d) As given that,
Lm Lsh
N2 or R = R
The inductance of primary is ( L) = P ...(i) m sh
R
where, R is reluctance of magnetic path. i.e., Tm = Tsh ( T ® Time constant)
As we know that, For, independent of frequency :

l 2pR é 2R ù æL ö
R= = =ê 2ú Tm = ç m ÷
mA m[pr ] ë mr û
2 [Q l = 2pR, A = pr 2 ] è Rm ø
By putting the value of R in equation (i)
0.1´10-3
= = 2 m sec.
é ù 50 ´10-3
ê 2 ú Hence, option (a) is correct.
N P ú é N P2 mr 2 ù
\L = ê = 28. (a) As we know that,
ê é 2R ù ú ê 2R ú
êê 2 úú ë û For (LL) fault, without Zf :
êë ë mr û úû é 3Eg ù
If = ê ú
We also know that XL = WL ë Z1 + Z 2 û
æ Vö Ans, (LL) fault, with Zf :
ç\ X L = ÷
è Iø é ù
3Eg
If1 = ê ú
æV ö êë Z1 + Z 2 + Z f úû
çè ÷ø = wL
I As given that,
æV ö é N P2 mr 2 ù I f 1 = K (I f ) ...(i)
çè ÷ø = w ê ú
I ë 2R û By putting the value of (If) and (If1) in equation (i).
é 3Eg ù é 3Eg ù
é wN P2 mr 2 I ù é mIr 2 N P2 w ù ê ú=Kê ú
R = ê ú=ê ú
So, êë Z1 + Z 2 + Z f úû ë Z1 + Z 2 û
ë 2V û ë 2V û
27. (a) As given that, ( Z1 + Z 2 ) = K ( Z1 + Z 2 + Z f )
Rm = 50 mW, I m = 1 Amp. ( Z1 + Z 2 ) = KZ1 + KZ 2 + KZ f
Lm = 0.1 mH, I = 10 Amp.
Z 2 (1 - K ) + Z1 (1 - K )
As we know that, = Zf
K
é Required (I ) ù
Scaling factor (m) = ê ú æ ( Z + Z 2 )(1 - K ) ö
Zf = ç 1
ë existing (Im) û K ÷
è ø
10 29. (c) As given that,
Þm=
1 The two bus power system network with given loads
as shown in figure
m = 10
We also know that (VS = 1.0 Ðd ) (VR = 1.0 Ð0 )
The required value shunt coil resistance is : PS j 0.1 PR
G1 G2
-3 QS QR
é R ù é 50 ´10 ù (20 + jQG1) (15 + jQG2)
Rsh = ê m ú = ê ú
Qloss
ë m - 1û ë 10 - 1 û
(15 + j5) (20 + j10)
Þ Rsh = 5.55 mW
17
We know that,
ép ù
Power transfer equation is Þ d = ê - 0.0219 ú = 0.5017 radian
ë6 û
é | V || V |sin d ù
PR = ê S R ú ...(i) 180
XS Þ d1 = 0.5017 ´
ë û p
From active power balance at bus 2,
d1 = 28.740
PR + PG 2 = PD 2
31. (b) As given that,
PR = (20 - 15) = 5 Pu
So, from equation (i) L=1H

é1´1´ sin d ù I(S)


5= ê +
ë 0.1 úû 5V + (t= 0) V (t) or
– C = 1F
S – æ1ö
sin d = 0.5 V (S ) = ç ÷
d = 30° èSø
We also know that,
é 5/ S ù 5
é V 2 ù éV V ù I (S ) = ê ú= 2
Qs = ê S ú - ê S R cos d ú ë S + (1/ S ) û ( S + 1)
êë X L úû ë X L û
I (t ) = 5 Sint
é 1 2
1´ 1 ù Similarly,
=ê - ´ cos 30° ú = 1.34 Pu
ë (0.1) 0.1 û é 1/ S ù
V (S ) = ê ú [5 / S ]
Similarly, ë S + 1/ S û

ù éV ù
2
éV V 5 é 5 ù é 5S ù
QR = ê S R (cos d) ú - ê R ú = =ê ú-ê 2 ú
ë XL û êë XL úû 2
S (S + 1) ë S û ë S + 1û

é1´1 (1)2 ù Þ V (t ) = 5(1 - cos t )


=ê (cos 30°) - ú = -1.34 Pu time (t) i(t) V(t)
ëê 0.1 0.1 ûú
0 0 0
Now, Q loss = QS – QR = 1.34 – (–1.34) = 2.68 Pu.
Qgain at (G1) = Qload + QS p
5 5
QG = 5 + 1.34 = 6.34 Pu 2
p 0 10
Similarly, Qgain at (G2) = (Qload – QR)
= 10 + 1.34 3p
–5 –5
= 11.34 Pu 2
30. (c) As given that, 2p 0 0

P = 0.8, d0 = 30°, i(t)

P (d) = 1.4 sin d + 0.15 sin 2d = 0.8


p
Þ P (d ) = 1.4sin d + 0.15sin 2d - 0.8 = 0 t=
2
5
éd ù
P '(d) = ê ( P(d)) ú
ëdd û
Þ P '(d) = 1.4 cos d + 0.30 cos 2d - 0 t=p
t=0
P '(d) = 1.4 cos d + 0.30 cos 2d t = 2p (5, 0) 10 V (t)
By using Newton Rephson Method :
é P (d ) ù
d1 = d 0 - ê ú
ë P '(d) û at ( d= d0 = 30°) –5
3p
t=
2
é p [1.4 sin 30° + 0.15 sin 60° - 0.8] ù
=ê - Hence, the graph between i(t) and V(t) is a circle of
ë6 [1.4 cos 30° + 0.30 cos 60°] úû radius 5 and having a center (5, 0).
18
32. (a) By taking laplace of both sides in equation (ii)
j 9W j 9W SX 2 ( S ) - X 2 (0) = X 1 (S ) - X 2 (S )
X 2 ( S )( S + 1) = X 2 (0) + X 1 ( S ) ...(iv)
Now, by using final value theorem
j5W j5W
X 1 f = lim X 1 (t ) = lim S × X 1 (S )
Zeq t ®0 S ®0
–j1W –j1W
é X (0) + X 2 ( S ) ù
= lim ê 1 ú =0
S ®0 ë ( S + 1) û
X 2 f = lim X 2 (t ) = lim ( S × X 2 ( S ))
t ®¥ S ®0
Z1 Z1 Z1
é X (0) + X 1 ( S ) ù
= lim ê 2 ú
S ®0 ë ( S + 1) û
Z2 Z2
Zeq Hence, X1 f = X 2 f < ¥
34. (a) As given that, the characteristic equation :
S 7 + S 6 + 7 S 5 + 14S 4 + 31S 3 + 73S 2 + 25 S + 200 = 0

7
S 1 7 31 25
A Z1 Zeq
6
S 1 14 73 200
5 Sign
Z2 S –7 –42 –175 0
Zeq change
4
S +8 +48 +200 0
B 3
S 1 3 0 0
Z eq = Z1 + ( Z 2 || Z eq ) S
2
24 200 0 0
é Z 2 * Zeq ù 1 Sign
= Z1 + ê ú S –128 0 0 0 change
ëê Z 2 + Zeq ûú 0
S 200 0 0 0
Given, Z1 = j 9W
Z 2 = j 5W - j1W = j 4W Now, Auxilary equation is
A(S) = 8S4 + 48S2 + 200
é j 4* Z eq ù
Z eq = j 9 + ê ú dA( S )
= (32 S 3 + 96 S )
êë j 4 + Z eq úû dS
Z eq (-4 j + Z eq ) = j9( j 4 + Z eq ) + j 4Z eq Two sign change above auxillary equation = 2 Poles
in R.H.S.
By solving it :- Two sign change below auxilary equation, so,
Z eq = j12 No. of sign changes = 4 (Symmetric roots about
origin)
33. (c) Given : So, the characteristics equation is having two Poles
dx1 (t ) in the R.H.S., which are having symmetrical Pole in
= X 2 (t ) - X 1 (t ) ...(i) L.H.S.
dt
There will be no Pole on imaginary axis.
dx2 (t ) So, total left hand Pole = 7 – 4 = 3.
= X 1 (t ) - X 2 (t ) ...(ii)
dt
é Z2 ù é Z2 ù
By taking laplace of both sides in equation (i) 35. (b) f (Z ) = ê 2 2 ú = ê 2 2 ú
SX 1 (S ) - X 1 (0) = X 2 ( S ) - X1 ( S ) ëê ( Z - 3Z + 2) ûú ëê ( Z - (2 + 1) Z + 2) ûú

X 1 (S )( S + 1) = X 1 (0) + X 2 (S ) ...(iii) é Z2 ù
=ê 2 2ú
êë ( Z - 2Z - Z + 2) úû
19
é Z2 ù DA = QB QC
=ê 2ú
êë ( Z ( Z - 1) - 2( Z - 1)) úû CLK (D A=QBQ C) DB=QA D C=QB Q A QB QC
(initial) (initial) (initial)
=0 =0 =0
é Z2 ù 1 0 0
=ê 2ú
1 1 0 0

ëê ((Z - 1)(Z - 2)) ûú 2 1 1 0 1 1 0

Z = 1 and 2 are two poles of order 2, both lies inside 3 1 1 1 1 1 1


the circle. 4 0 1 1 0 1 1
Now, residue of f(Z) at (Z = 1). 5 0 0 1 0 0 1
So,
6 1 0 0 1 0 0 Repeated
é 1 d Z2 ù 7 1 1 0 1 1 0
Res f ( Z ) = ê lim ( Z - 1) 2 ´ 2 2 ú
Z =1
ë (1)! Z ®1 dZ ( Z - 1) ( Z - 2) û At 6th clock pulse counter reaches to the value of
1st clock pulse so it is MOD5 counter.
é d æ Z2 öù Hence, it can be used for dividing the input frequency
= lim ê çç 2
÷÷ ú by 5.
Z ®1 ê dZ
ë è ( Z - 2) ø ûú 37. (b) As given that,
é (Z - 2)2 2 Z - Z 2 ´ 2( Z - 2) ù F = m0 + m2 + m3 + m5 + m7
= lim ê ú Don't care = (1, 4, 6)
Z ®1
ë (Z - 2)4 û
=4 F = å m(0, 2, 3, 5, 7) + d (1)
Now, Residue of f(Z) at Z = 2
So, BC
A BC BC BC BC
é 1 d ( Z - 2) 2 ´ Z 2 ù 1
Res f ( Z ) = ê lim A 1 X 1
2 2ú 0 1 3 2
Z =2
ë (1)! Z ® 2 dZ ( Z - 1) ( Z - 2) û 1
A 1
4 5 7 6
é d æ Z2 öù
= lim ê çç 2
÷÷ ú
Z ® 2 ê dZ
ë è (Z - 1) ø ûú F = ( A + C)
38. (a) As given that,
é ( Z - 1)2 (2Z ) - Z 2 2(Z - 1) ù
= lim ê ú
Z ®2 ê ( Z - 1)4 úû ì t , -1 £ t £ 1 ü
ë X 1 (t ) = í ý
î 0, otherwiseþ
é4 -8ù
=ê ú = -4. ì1- | t | -1 £ t £ 1 ü
ë 1 û X 2 (t ) = í ý
\ Thus, By Cauchy Residue Theorem : î 0, otherwise þ
For X1(t) :
I = ò f (Z ) dZ = 2pi[ R1 + R2 ] = 2pi[4 - 4] = 0.
C X1(t)
36. (b) As given circuit 1

t
–1 1
DA QA DB QB DC QC fout
C C C X 1 (n) = [1, 0.75, 0.5, 0.25, 0, 0.25, 0.5, 0.75, 1]
(Q T = 0.25)
fIN Now, Energy of X1(n)
¥
= å | X 1 ( n) |2 = [12 + 0.752 + 0.52 + 0.252 ] ´ 2
n = -¥
QA QB QC
= 3.75 Joule
CLK
20
X2(t) [ X (t )]t =1 = e-10
By taking log both side in equation (i)
1 ln ( x (t ) ) = ln(t × e -10t u (t ))

| ln( x(t )) |t = 1 = | ln t + ( -10t (ln e e) u (t )) |

t =| ln 1 - 10 ´ 1 ´ 1| = | 0 - 10 | = | -10 | = 10.0
–1 1 41. (90) As given circuit :-
X X X
10kW E
a
X 2 (n) = [0, 0.25, 0.5, 0.75, 1, 0, 0.75, 0.5, 0.25, 0] VBE=0.7
B
15V +
¥ 10kW
So, Energy of X 2 (n ) = å | X 2 ( n ) |2

1kW
n = -¥ +
10.7V Rth
2 2 2 IB
= [(0.25) + 0.5 + 0.75 ] ´ 2 + 1 –
= 0.875 × 2 + 1 = 2.75 Joule. b
Hence, we can see clearly, energy of X1(n) is greater By using KVL
than the energy of X2(n). +10.7 - 0.7 - 10 KI B - 1K (b + 1) I B = 0
39. (d) As given that,
X (t ) = [(t - 1)u (t - 1)] - [(t - 2)u (t - 2]
10 é 10m ù
IB = =
(10 K + 101K ) êë111 ´ 103 úû
-[(t - 3)u (t - 3)] + [(t - 4)u (t - 4)]
This equation we can written as : æ 101 ´ 10m ö
I E = (1 + b) I B = ç (Q b = 100)
So, X (t ) = r (t - 1) - r (t - 2) - r (t - 3) - r (t - 4) è 111 ´ 103 ÷ø
Case I : When ab terminate is open circuit,
r (t ) = tu (t )
VOC = I E ´ 1K
X(t)
101´ 10m ´103
VOC =
1 111´103
1
VOC = 9.09
(t – 1) (t– 4)
I II III Case II : When a - b terminals is short circuit.

1 2 3 4 10W E –
+
Now, energy E X(t) 15V –
B 0.7
¥
+
1kW
= ò | X (t ) | dt 2
10kW
¥

2 3 4 IB IN
= ò (t - 1) 2 dt + ò (1) 2 dt + ò (t - 4) 2 dt +
10.7V – 10.7 V
1 2 3

1 1 2 5
= + 1+ = +1 = . P
3 3 3 3
40. (10) As given that So, there will be no current in 1kW resistor hence,
(IN = IE).
é 1 ù By using KVL :-
X (w ) = ê 2ú
ë (10 + j w ) û +10.7 - 0.7 - 10 K ( I B ) = 0
By taking inverse fourter transform,
é10.7 - 0.7 ù
X (t ) = t × e -10t u (t ) ...(i) IB = ê ú = 1 m Amp
ë 10k û
Then, [ X (t )]at t =1 = 1 ´ e - ´ ´ u (1)
10 1
I SC = (1 + b) I B = (1 + 100) ´1 m Amp.

= 1´ e -10 ´ 1 I SC = 101 m Amp.


21
Hence, the thevenin equivalent resistance between 2
9 x - 14 x + 5 = 0
AB,
5
é V ù éV ù 9.09 x = 1 and
Rth = ê th ú = ê oc ú = = 90W 9
I I
ëê SC ûú ê sc ú 101 ´ 10-3
Now, f ''( x ) = (18x - 14)
42. (0) As given that,
So, at x = 1 f ''(1) = 18 ´1 - 14 = 4
x2 + y2 = 9
f '(1) > 0, so x = 1 point of local minimum.
y2 = 9 - x2
5
y = (9 - x 2 ) at x =
9
1 æ5ö æ 5 ö
dy = ( -2 x dx) f '' ç ÷ = ç18 ´ - 14 ÷
2
2 9- x è9ø è 9 ø

-x æ5ö
dy = dx f ' ç ÷ = -4
9 - x2 è9ø
æ5ö 5
Now, I = Ñò ( y 2 + 2 yx) dx + (2 xy + x 2 ) dy f '' ç ÷ < 0 , so x = is point of (local) maximum.
9
è ø 9

ò [( f1 dx)] + Ñò [( f2 dy )] æ 5ö
So, f (x) at ç x = ÷
è 9ø
(Q y = )
0

Ñò f1 (dx ) = Ñ
ò ( y + 2 yx) dx
2
C
9 - x2 3 2
3 æ5ö æ5ö æ5ö
= 3 ç ÷ - 7 ç ÷ + 5 ç ÷ + 6 = 7.1
è9ø è9ø è9ø

ò [(9 - x ) + 2x (9 - x )] dx
2 2
At x = 0, f (x) = 6
C
But maximum value can also occur at where points,
0
é 1 ù so, f (2) = 3(2)3 - 7(2) 2 + 5(2) + 6 = 12.
+1
0 ê x 3 (9 - x 2 ) 2 x3 ú
Ñò f1 dx = ê9 x - -
ê 3 æ1 ö
- 2 ú = -18

Hence, absolute maximum value = 12.
44. (1) As we know that characteristic equation
3
ê çè + 1 ÷ø ú
ë 2 û3 A = | A - lI |= 0
Similarly As given that,
0 0
é (- x ) ù é 1 0 -1ù
Ñò f 2 dy = Ñ
ò ((2 x 9 - x ) + x ) êê
2 2
ú dx
3 3 ë 9 - x ûú
2 A = êê -1 2 0 úú
ëê 0 0 -2 ûú
é 0 æ x3 ö ù
= ê- Ñ
ò ç ÷ dx ú
So,
êë 3 è 9 - x 2 ø úû
é(1 - l) 0 -1 ù
2
By putting, (9 - x ) = Z Þ 0 - 2 x = dZ3 ê
Þ ê -1 (2 - l) 0 úú = 0
3
êë 0 0 (-2 - l) úû
é (9 - Z ) æ -dZ ö ù é 3 æ dZ ö ù
=- Ñò ê
ë Z
çè
2
÷ø ú = +18 êQ x = ç - ÷ ú
û ë è 2 øû Þ (1 - l)[(2 - l)(-2 - l)] - 1[0 - 0] = 0
z= 0
So, Þ l = 1, 2, - 2
By Cayley Hamilton theorem,
I= Ñò C f1 ( x) + Ñò C f2 ( x) = -18 + 18 = 0 A3 - A2 - 4 A + 4I = 0
43. (12) As given that
Now, A3 - A2 - 4 A + 5I = I
3 2
f ( x ) = 3x - 7 x + 5x + 6
So, B = I (Q B = A3 - A2 - 4 A + 5I (given))
2
f '( x) = (9 x - 14 x + 5)
Þ| B |=| I |= 1
So, critical points are f '(x) = 0
22
45. (2.5) As given that
C1 = 60 P.f. é K ù
G(S ) H (S ) = ê 2 ú
Ceq = 86 P.f. ë ( S + 1) ( S + 2) û
C1 C2 For, unit step input (KP)
K P = lim[G ( S ) × H ( S )]
S ®0

Îr é ù
Air K
= lim ê 2 ú
S ® 0 ( S + 1) ( S + 2)
ë û

K
KP =
d/2 d/2 2
Now, the steady state error do unit step input is
é C1C2 ù given by,
So, Ceq = ê C + C ú
ë 1 2û 1
eSS = = 0.2
(1 + K P )
éC1 =Î0 A / (d / 2) ù
ê ú
Where, êC = Î A = Î0 Îr A ú , [QÎ=Îo Îr ] (1 + K P ) =
1
= 5
êë 2
d/2 d / 2 úû 0.2
é Î0 A éÎ0 Îr A ù ù æ Kö æ Kö
ê d /2´ê d /2 úú ç1 + ÷ = 5 çQ K P = ÷
ë ûú è 2ø è 2ø
So, Ceq = ê
ê Î0 Îr A Î0 A ú
ê d /2 + d /2 ú K
ë û Þ = 5 -1 Þ K = 4 ´ 2 = 8
2
é Î0 A éÎ0 Îr A ù ù 47. (5.77)
ê d /2´ê d /2 úú As given circuit :
Ceq = ê ë ûú
ê Î0 A ú a
ê d / 2 (Îr +1) ú
I1
ë û
é 2Î Î Aù R
Ceq = ê 0 r ú
ë d (Îr +1) û
d
n
é 2C Î ù I2
Þ Ceq = ê 1 r ú
ë (Îr +1) û j10
Ceq 2 Îr
Þ =
C1 (Îr +1) –j10
86 2 Îr
Þ = I3
60 (Îr +1)
Þ 86(Îr +1) = 120 Îr
Þ 120 Îr -86 Îr = 86 I4

84
Þ 34 Îr = 84 ÞÎr = = 2.53 By using KCL at node (d), we get,
34 ur uur uur uur
I1 + I 3 + I 4 = I 2
Îr = 2.53 ur ur ur
V ad V cd V bd uur
46. (8) As given that, + + = I2
R - j10 j10
Input is unit step = 1
Output is steady state = 0.8 100 100Ð - 240° 100Ð - 120° uur
Ð0° + + = I2
The steady state error (eSS) = 1 – 0.8 = 0.2. R 10Ð – 90° 10Ð90°
Also, given that transfer function uur 100Ð0°
Þ I2 = + 10Ð - 150° + 10Ð210°
R
23
uur So, average voltage across inductor is zero
As given that I 2 = 0 ,
V0 = ( Eb + I 0 ´ Ra + 0)
é100 ù
0= ê - 8.66 + j5 - 8.66 - j5ú Vm
ë R û (3 + cos a ) = 80 + (20 ´ 2) + 0
2p
100
- 17.32 = 0 80p
R Þ (3 + cos a) = 120°
2p
100
ÞR= Þ 3 + cos a = 3 Þ cos a = 0
17.32
a = 90°
R = 5.77W 50. (12.5)
48. (10) As given that, As given that,
V(t) = 5 – 10cos(wt + 60°) V
or, V(t) = 5 + 10 cos(wt – 120°) D = 0.5, R = 1 W , vdC = 100
I (t ) = 5 + X cos(wt ) A We know that,
V0 = DVdC
= 5 + X cos(wt + 0°)
V0 = 0.5 ´100 = 50 Volt
Pavg = 0
Now, Average Power : æ 50 ö 50
I 0 R = 50 Þ I 0 = ç ÷ = = 50 Amp
1
T è Rø 1
T ò0
Pavg = V (t ) × I (t ) dt Since all components are ideal. So, input power is
equal to the output power.
We know that, the standard form of (Pavg) is (VS I S1 + VS I S2 = V0 I 0 )
V I Since the duty cycles of both the converters are
Pavg = V0 i0 + 1 × 1 cos(fV1 - fi1 )
2 2 identicals.
V2 I2 ( I S1 = I S2 )
+ × cos(fV2 - fi2 ) + ......
2 2 (VS I S1 + VS I S1 ) = V0 I 0
where,
VS *2 I S1 = V0 I 0
V0 = 5, V1 = 10, fV1 = -120° , i0 = 5, I1 = X, fi1 = 0°
So, 100 ´ 2 I S1 = 50 ´ 50
10 x I S1 = 12.5 Amp
Pavg = 5 ´ 5 + ´ cos( -120° - 0°)
2 2 51. (97.36)
As given that,
10 x
0 = 25 + cos(-120°) R1 = 0.3 W, R2 = 0.02 W
2
3f Transformer rating (S) = 900 KVA,
æ -1 ö D connected high voltage rating = 3 KV
Þ 0 = 25 + 5 x ç ÷
è 2 ø Y connected low voltage rating = 3 KV
5 So, the current in high voltage winding at full load
Þ x = 25
2 é 900 ´ 1000 ù
( I hv ) = ê ú
25 ´ 2 ë 3 ´ 3 ´ 1000 û
Þx= = 10.
5 I hv = I1 = 100 Amp
x = 10 3
49. (90) As given that, aPh = Þ aPh = 3
3/ 3
V We know that
V0 = m (3 + cos a)
2p 2
R(eq ) = R1 + aPh × R2 = 0.3 + (3)2 ´ 0.02 = 0.48 W
Ra = 2W, Vm = 80p volts At full load, unity power factor
Battery voltage Eb = 80 volt Pout = 900 × 1 = 900 kW
Power delivered to loss less batery (EbI0) = 1600 W Ploss = Cu loss (3 phase) + Core loss
æ 1600 ö = (3I12 Req + 10 KW )
I0 = ç ÷
è Eb ø = (3 ´ (100)2 ´ 0.48 + 10 KW ) = 24.4 KW
æ 1600 ö \ Transformer efficiency at full load,
I0 = ç ÷ = 20 A
è 80 ø é Pout ù 900
h= ê ú= = 97.36%.
ë Pout + Ploss û 900 + 24.4
24
52. (825) So, at same voltage
As given that, Eb2 = V - I a2 ( Ra + R f )
Terminal voltage (Vt) = 200 V, N = 1000
= 200 – 12 × 1 = 188 Volt.
I a1 = 10 Amp , ( Ra + R f ) = 1 Now, we also know that,
R f= 1W
éE ù
Nµê bú
10A ë fû
200V
N1 Eb1 é f2 ù é f 2 I a2 ù
= ´ê ú êQ = ú
N 2 Eb2 ë f1 û
ëê f1 I a1 úû
From the above circuit diagram :
1000 190 é I a2 ù é190 12 ù
Eb1 = V - I a1 ( Ra + R f ) = ´ê ú = ´
N2 188 êë I a1 ûú êë188 10 úû
= 200 – 10 × 1 (Q Ra + R f = 1)
é1000 ´188 ´10 ù
Eb1 = 190 V N2 = ê ú = 824.56 ; 825 rpm
ë 190 ´ 12 û
Also given that,
load torque increased by 44% 53. (12) As given that,
so, T2 = 1.44T1 Vin = 50 V, Vout = 150 V
We know that, L = 5 mH, D = 0.4
VL
T µ I a2
2 iL (Imax)
T æ Ia ö 50 Volt
Þ 2 =ç 2÷
T1 çè I a1 ÷ø
DT bT
æ T ö 1.44T1 O Ton
Þ I a2 = ç 2 ÷ ´ I a1 = ´10 –100 Volt
ç T ÷ T1
è 1ø
Þ I a2 = 12 A
So, the inductor discharges during 0.4T to 0.6T and
power transferred from Vin to Vout. The given circuit is a boost converter
Thus, the value of Vin : We know that,
For continuous condition
dl DI
Vin = L = L é V ù
dt ton V0 = ê in ú
ë (1 - D) û
æV ´t ö
DI = ç in on ÷ 50 50
è L ø So, V0 = = = 83.33 V
1 - 0.4 0.6
\ ton = D * T Now, output voltage is
1 é b ù
T= = 200 m sec. Vout = ê
f ú Vin
ëb - D û
D = 0.4 (given)
b
So, ton = 200 × 10–6 × 0.4 Þ 150 = ´ 50
ton = 80 m sec. (b - 0.4)
50 ´ 80 ´10-6 Þ 2b = 1.2 Þ b = 0.6
Thus, DI = = 0.8 Amp. 54. (0.106)
5 ´10-3
As given that,
DI = I max - I min. (Q I min. = 0)
R1 = R '2 = X l1 = X 'l2 = 12W
DI = I max.
X M = 240W
I max. = 0.8 Amp.
At no load,
Hence, Power transferred from B1 to B2 or, for charging
and discharging period of Inductor Wm ; WS
1 R '2
= Vout ´ I max ( B - D ] So, »¥ (Q S = 0)
2 2S
1 Therefore, the equivalent circuit can be drawn as:
= 150 ´ ´ 0.8 ´ (0.6 - 0.4)
2
= 15 × 0.8 = 12 W.
25
Now,
12W j12W
é ( j120)(3 + j 6) ù
Z eq = 12 + j12 + j120 + ê ú
R1 ë (3 + j126) û
X21 Z eq = 12 + j132 + 2.72 + j 5.78
R '2 / 2 S Z eq = 14.72 + j137.78
X = 138.561 Ð83.9
j m
2 X'
VÐ 0° j l2 at No load, Power sector ( Pf ) = cos f = cos83.9
2
P. f = 0.106
55. (50) As given equivalent circuit is
+ L i(t)
R '2 a
Xm 2 (2-S)
j 100W
2 V(t)
Zeq Þ 100mf
Xl '2
j
2 b

where, L = j100 L (Q w = 100 radian/sec)
é ( - j100) ´ 100 ù
Z eq = j100 L + ê ú
ë (100 - j100) û
é - j104 ù
= j100 L + ê ú
12W j12W
ë (100 - j100) û
é - j104 ´ (100 + j100) ù
= [ j100 L] + ê ú
ë (100 - j100)(100 + j100) û
é 106 ù é æ 106 öù
=ê 2 2 ú
+ j ê100 L - ç 2 2 ÷ú
æ 240 ö ë (100 + 100 ) û ëê è 100 + 100 ø ûú
jç ÷
è 2 ø Z eq = 50 + j (100 L - 50)
V Ð0° Now, at resonance imaginary component of Zeq = 0
So, j (100 L - 50) = 0
Þ
Hence, Z eq = 50

12
æ 240 ö 4
jç ÷
è 2 ø
j12
2

Вам также может понравиться